Trouble understanding Poisson Brackets












1












$begingroup$


I'm looking at page 94 here - I understand the definition of Poisson brackets at the top of the page (which uses summation convention) but I don't get why the calculations in (4.61) are true. I'm substituting in the generalised coordinate $q_i$ as $f$ and the generalised momentum $p_i$ as $g$, but I must be doing something wrong - do the subscripts in (4.61) not correspond to the subscripts in the definition?










share|cite|improve this question









$endgroup$












  • $begingroup$
    What is $q_i$? In the link you provided, authors claim ${q_i, q_j}=0$, where are $q_i, q_j$ defined?
    $endgroup$
    – 5xum
    Aug 25 '15 at 12:02










  • $begingroup$
    $q_i$ for $i=1,...,n$ are the generalised coordinates used in the Lagrangian/Hamiltonian formulations
    $endgroup$
    – jl2
    Aug 25 '15 at 12:07












  • $begingroup$
    That's not much of a definition. Are they solutions to some differential equation or something like that?
    $endgroup$
    – 5xum
    Aug 25 '15 at 12:09










  • $begingroup$
    You need to use the Hamiltonian equation.
    $endgroup$
    – user40276
    Aug 25 '15 at 12:11










  • $begingroup$
    @5xum: It's part of the generic framework which goes: Suppose we have some abstract space $X$ of states parameterized by $q_1,q_2,ldots,q_n$. Then the operands to the Poisson bracket are functions $Xtimes Ytimes mathbb Rtomathbb R$, and the notation $q_i$ is abused to also mean the function that simply returns the $i$'th coordinate of its $X$ input.
    $endgroup$
    – Henning Makholm
    Aug 25 '15 at 12:13


















1












$begingroup$


I'm looking at page 94 here - I understand the definition of Poisson brackets at the top of the page (which uses summation convention) but I don't get why the calculations in (4.61) are true. I'm substituting in the generalised coordinate $q_i$ as $f$ and the generalised momentum $p_i$ as $g$, but I must be doing something wrong - do the subscripts in (4.61) not correspond to the subscripts in the definition?










share|cite|improve this question









$endgroup$












  • $begingroup$
    What is $q_i$? In the link you provided, authors claim ${q_i, q_j}=0$, where are $q_i, q_j$ defined?
    $endgroup$
    – 5xum
    Aug 25 '15 at 12:02










  • $begingroup$
    $q_i$ for $i=1,...,n$ are the generalised coordinates used in the Lagrangian/Hamiltonian formulations
    $endgroup$
    – jl2
    Aug 25 '15 at 12:07












  • $begingroup$
    That's not much of a definition. Are they solutions to some differential equation or something like that?
    $endgroup$
    – 5xum
    Aug 25 '15 at 12:09










  • $begingroup$
    You need to use the Hamiltonian equation.
    $endgroup$
    – user40276
    Aug 25 '15 at 12:11










  • $begingroup$
    @5xum: It's part of the generic framework which goes: Suppose we have some abstract space $X$ of states parameterized by $q_1,q_2,ldots,q_n$. Then the operands to the Poisson bracket are functions $Xtimes Ytimes mathbb Rtomathbb R$, and the notation $q_i$ is abused to also mean the function that simply returns the $i$'th coordinate of its $X$ input.
    $endgroup$
    – Henning Makholm
    Aug 25 '15 at 12:13
















1












1








1





$begingroup$


I'm looking at page 94 here - I understand the definition of Poisson brackets at the top of the page (which uses summation convention) but I don't get why the calculations in (4.61) are true. I'm substituting in the generalised coordinate $q_i$ as $f$ and the generalised momentum $p_i$ as $g$, but I must be doing something wrong - do the subscripts in (4.61) not correspond to the subscripts in the definition?










share|cite|improve this question









$endgroup$




I'm looking at page 94 here - I understand the definition of Poisson brackets at the top of the page (which uses summation convention) but I don't get why the calculations in (4.61) are true. I'm substituting in the generalised coordinate $q_i$ as $f$ and the generalised momentum $p_i$ as $g$, but I must be doing something wrong - do the subscripts in (4.61) not correspond to the subscripts in the definition?







partial-derivative mathematical-physics classical-mechanics






share|cite|improve this question













share|cite|improve this question











share|cite|improve this question




share|cite|improve this question










asked Aug 25 '15 at 11:56









jl2jl2

500313




500313












  • $begingroup$
    What is $q_i$? In the link you provided, authors claim ${q_i, q_j}=0$, where are $q_i, q_j$ defined?
    $endgroup$
    – 5xum
    Aug 25 '15 at 12:02










  • $begingroup$
    $q_i$ for $i=1,...,n$ are the generalised coordinates used in the Lagrangian/Hamiltonian formulations
    $endgroup$
    – jl2
    Aug 25 '15 at 12:07












  • $begingroup$
    That's not much of a definition. Are they solutions to some differential equation or something like that?
    $endgroup$
    – 5xum
    Aug 25 '15 at 12:09










  • $begingroup$
    You need to use the Hamiltonian equation.
    $endgroup$
    – user40276
    Aug 25 '15 at 12:11










  • $begingroup$
    @5xum: It's part of the generic framework which goes: Suppose we have some abstract space $X$ of states parameterized by $q_1,q_2,ldots,q_n$. Then the operands to the Poisson bracket are functions $Xtimes Ytimes mathbb Rtomathbb R$, and the notation $q_i$ is abused to also mean the function that simply returns the $i$'th coordinate of its $X$ input.
    $endgroup$
    – Henning Makholm
    Aug 25 '15 at 12:13




















  • $begingroup$
    What is $q_i$? In the link you provided, authors claim ${q_i, q_j}=0$, where are $q_i, q_j$ defined?
    $endgroup$
    – 5xum
    Aug 25 '15 at 12:02










  • $begingroup$
    $q_i$ for $i=1,...,n$ are the generalised coordinates used in the Lagrangian/Hamiltonian formulations
    $endgroup$
    – jl2
    Aug 25 '15 at 12:07












  • $begingroup$
    That's not much of a definition. Are they solutions to some differential equation or something like that?
    $endgroup$
    – 5xum
    Aug 25 '15 at 12:09










  • $begingroup$
    You need to use the Hamiltonian equation.
    $endgroup$
    – user40276
    Aug 25 '15 at 12:11










  • $begingroup$
    @5xum: It's part of the generic framework which goes: Suppose we have some abstract space $X$ of states parameterized by $q_1,q_2,ldots,q_n$. Then the operands to the Poisson bracket are functions $Xtimes Ytimes mathbb Rtomathbb R$, and the notation $q_i$ is abused to also mean the function that simply returns the $i$'th coordinate of its $X$ input.
    $endgroup$
    – Henning Makholm
    Aug 25 '15 at 12:13


















$begingroup$
What is $q_i$? In the link you provided, authors claim ${q_i, q_j}=0$, where are $q_i, q_j$ defined?
$endgroup$
– 5xum
Aug 25 '15 at 12:02




$begingroup$
What is $q_i$? In the link you provided, authors claim ${q_i, q_j}=0$, where are $q_i, q_j$ defined?
$endgroup$
– 5xum
Aug 25 '15 at 12:02












$begingroup$
$q_i$ for $i=1,...,n$ are the generalised coordinates used in the Lagrangian/Hamiltonian formulations
$endgroup$
– jl2
Aug 25 '15 at 12:07






$begingroup$
$q_i$ for $i=1,...,n$ are the generalised coordinates used in the Lagrangian/Hamiltonian formulations
$endgroup$
– jl2
Aug 25 '15 at 12:07














$begingroup$
That's not much of a definition. Are they solutions to some differential equation or something like that?
$endgroup$
– 5xum
Aug 25 '15 at 12:09




$begingroup$
That's not much of a definition. Are they solutions to some differential equation or something like that?
$endgroup$
– 5xum
Aug 25 '15 at 12:09












$begingroup$
You need to use the Hamiltonian equation.
$endgroup$
– user40276
Aug 25 '15 at 12:11




$begingroup$
You need to use the Hamiltonian equation.
$endgroup$
– user40276
Aug 25 '15 at 12:11












$begingroup$
@5xum: It's part of the generic framework which goes: Suppose we have some abstract space $X$ of states parameterized by $q_1,q_2,ldots,q_n$. Then the operands to the Poisson bracket are functions $Xtimes Ytimes mathbb Rtomathbb R$, and the notation $q_i$ is abused to also mean the function that simply returns the $i$'th coordinate of its $X$ input.
$endgroup$
– Henning Makholm
Aug 25 '15 at 12:13






$begingroup$
@5xum: It's part of the generic framework which goes: Suppose we have some abstract space $X$ of states parameterized by $q_1,q_2,ldots,q_n$. Then the operands to the Poisson bracket are functions $Xtimes Ytimes mathbb Rtomathbb R$, and the notation $q_i$ is abused to also mean the function that simply returns the $i$'th coordinate of its $X$ input.
$endgroup$
– Henning Makholm
Aug 25 '15 at 12:13












1 Answer
1






active

oldest

votes


















0












$begingroup$

${q_i,q_j}=frac{partial q_i}{partial q_k}frac{partial q_j}{partial p_k} - frac{partial q_j}{partial q_k}frac{partial q_i}{partial p_k}=delta_{ik}cdot 0 - delta_{jk}cdot 0=0$






share|cite|improve this answer









$endgroup$













    Your Answer





    StackExchange.ifUsing("editor", function () {
    return StackExchange.using("mathjaxEditing", function () {
    StackExchange.MarkdownEditor.creationCallbacks.add(function (editor, postfix) {
    StackExchange.mathjaxEditing.prepareWmdForMathJax(editor, postfix, [["$", "$"], ["\\(","\\)"]]);
    });
    });
    }, "mathjax-editing");

    StackExchange.ready(function() {
    var channelOptions = {
    tags: "".split(" "),
    id: "69"
    };
    initTagRenderer("".split(" "), "".split(" "), channelOptions);

    StackExchange.using("externalEditor", function() {
    // Have to fire editor after snippets, if snippets enabled
    if (StackExchange.settings.snippets.snippetsEnabled) {
    StackExchange.using("snippets", function() {
    createEditor();
    });
    }
    else {
    createEditor();
    }
    });

    function createEditor() {
    StackExchange.prepareEditor({
    heartbeatType: 'answer',
    autoActivateHeartbeat: false,
    convertImagesToLinks: true,
    noModals: true,
    showLowRepImageUploadWarning: true,
    reputationToPostImages: 10,
    bindNavPrevention: true,
    postfix: "",
    imageUploader: {
    brandingHtml: "Powered by u003ca class="icon-imgur-white" href="https://imgur.com/"u003eu003c/au003e",
    contentPolicyHtml: "User contributions licensed under u003ca href="https://creativecommons.org/licenses/by-sa/3.0/"u003ecc by-sa 3.0 with attribution requiredu003c/au003e u003ca href="https://stackoverflow.com/legal/content-policy"u003e(content policy)u003c/au003e",
    allowUrls: true
    },
    noCode: true, onDemand: true,
    discardSelector: ".discard-answer"
    ,immediatelyShowMarkdownHelp:true
    });


    }
    });














    draft saved

    draft discarded


















    StackExchange.ready(
    function () {
    StackExchange.openid.initPostLogin('.new-post-login', 'https%3a%2f%2fmath.stackexchange.com%2fquestions%2f1409032%2ftrouble-understanding-poisson-brackets%23new-answer', 'question_page');
    }
    );

    Post as a guest















    Required, but never shown

























    1 Answer
    1






    active

    oldest

    votes








    1 Answer
    1






    active

    oldest

    votes









    active

    oldest

    votes






    active

    oldest

    votes









    0












    $begingroup$

    ${q_i,q_j}=frac{partial q_i}{partial q_k}frac{partial q_j}{partial p_k} - frac{partial q_j}{partial q_k}frac{partial q_i}{partial p_k}=delta_{ik}cdot 0 - delta_{jk}cdot 0=0$






    share|cite|improve this answer









    $endgroup$


















      0












      $begingroup$

      ${q_i,q_j}=frac{partial q_i}{partial q_k}frac{partial q_j}{partial p_k} - frac{partial q_j}{partial q_k}frac{partial q_i}{partial p_k}=delta_{ik}cdot 0 - delta_{jk}cdot 0=0$






      share|cite|improve this answer









      $endgroup$
















        0












        0








        0





        $begingroup$

        ${q_i,q_j}=frac{partial q_i}{partial q_k}frac{partial q_j}{partial p_k} - frac{partial q_j}{partial q_k}frac{partial q_i}{partial p_k}=delta_{ik}cdot 0 - delta_{jk}cdot 0=0$






        share|cite|improve this answer









        $endgroup$



        ${q_i,q_j}=frac{partial q_i}{partial q_k}frac{partial q_j}{partial p_k} - frac{partial q_j}{partial q_k}frac{partial q_i}{partial p_k}=delta_{ik}cdot 0 - delta_{jk}cdot 0=0$







        share|cite|improve this answer












        share|cite|improve this answer



        share|cite|improve this answer










        answered Jan 17 at 12:16









        tommycauterotommycautero

        867




        867






























            draft saved

            draft discarded




















































            Thanks for contributing an answer to Mathematics Stack Exchange!


            • Please be sure to answer the question. Provide details and share your research!

            But avoid



            • Asking for help, clarification, or responding to other answers.

            • Making statements based on opinion; back them up with references or personal experience.


            Use MathJax to format equations. MathJax reference.


            To learn more, see our tips on writing great answers.




            draft saved


            draft discarded














            StackExchange.ready(
            function () {
            StackExchange.openid.initPostLogin('.new-post-login', 'https%3a%2f%2fmath.stackexchange.com%2fquestions%2f1409032%2ftrouble-understanding-poisson-brackets%23new-answer', 'question_page');
            }
            );

            Post as a guest















            Required, but never shown





















































            Required, but never shown














            Required, but never shown












            Required, but never shown







            Required, but never shown

































            Required, but never shown














            Required, but never shown












            Required, but never shown







            Required, but never shown







            Popular posts from this blog

            Mario Kart Wii

            What does “Dominus providebit” mean?

            Antonio Litta Visconti Arese